2014 dxdy logo

Научный форум dxdy

Математика, Физика, Computer Science, Machine Learning, LaTeX, Механика и Техника, Химия,
Биология и Медицина, Экономика и Финансовая Математика, Гуманитарные науки


Правила форума


В этом разделе нельзя создавать новые темы.

Если Вы хотите задать новый вопрос, то не дописывайте его в существующую тему, а создайте новую в корневом разделе "Помогите решить/разобраться (М)".

Если Вы зададите новый вопрос в существующей теме, то в случае нарушения оформления или других правил форума Ваше сообщение и все ответы на него могут быть удалены без предупреждения.

Не ищите на этом форуме халяву, правила запрещают участникам публиковать готовые решения стандартных учебных задач. Автор вопроса обязан привести свои попытки решения и указать конкретные затруднения.

Обязательно просмотрите тему Правила данного раздела, иначе Ваша тема может быть удалена или перемещена в Карантин, а Вы так и не узнаете, почему.



Начать новую тему Ответить на тему
 
 Putnam 2012 обобщение B6 - гипотезы
Сообщение06.12.2012, 18:34 
Заслуженный участник


08/04/08
8556
Задача В6 следующая (взял тут):
dm в сообщении #653395 писал(а):
B6
Пусть $p$ --- нечетное простое такое что $p\equiv 2\pmod{3}.$
Определим перестановку
$\pi$ остатков по модулю $p$ формулой $\pi(x)\equiv x^3\pmod{p}.$
Докажите, что перестановка $\pi$ четная если и только
если $p\equiv 3\pmod{4}.$

Как ее доказывать, я вообще не представляю и не понимаю, потому попытался обобщить. Получил следующие гипотезы:

1. Пусть $n$ нечетно, свободно от квадратов. Пусть $\pi_{n,d}(x)\equiv x^d\pmod n$ - перестановка $\mathbb{Z}_n^\times$ (это будет в точности для всех $d:\gcd(d,\varphi(n))=1$). Тогда $\pi$ - четная перестановка для всех $d$ $\Leftrightarrow$ в каноническом разложении $n$ число различных простых вида $p\equiv 1\pmod 4$ четно. В частности, если $n$ разлагается на произведение простых вида $p\equiv -1\pmod 4$, то все перестановки $\pi_{n,d}$ нечетны
В случае остальных $n$ имеют место как четные, так и нечетные перестановки, причем перестановок обеих типов одинаковое число (легко доказать одинаковость числа четных и нечетных перестановок в соответствующим случае).
2. Четность перестановки $x\to x^d\pmod n$ равна сумме четностей перестановок $x\to x^d\bmod {p_j}$, т.е. $s(\pi_{n,d})\equiv \sum\limits_{p\mid n}s(\pi_{p,d})\pmod 2$
3. Если $d\equiv 1\pmod 4$, то для любого $n$, свободного от квадратов, все . перестановки $\pi_{n,d}: x\tox^d\pmod n$ четны. В случае $d\equiv -1\pmod 4$ могут быть как четны, так и нечетны. Случай $d=3$ описан, при $d=5$ все перестановки $\pi_{n,5}$ четны, при $d=7$ (и прочих) описать затрудняюсь.

 Профиль  
                  
 
 Re: Putnam 2012 обобщение B6 - гипотезы
Сообщение06.12.2012, 19:38 
Модератор
Аватара пользователя


30/06/10
980
Как решать саму задачу, понятно: $\mathbb{Z}_p^*$ циклическая, поэтому в сущности задача про перестановку, заданную умножением на $3$ в $\mathbb{Z}_{p-1}$.

Только что дальше, не знаю. Единственное, что пока пришло в голову -- если не найдется такого $k$, что $3^k = -1\pmod {p-1}$, то перестановка четная. (Тогда циклы в перестановке очевидно разбиваются на пары.)

-- Чт дек 06, 2012 19:44:36 --

Второе, что пришло в голову -- если $\operatorname{ord}_{p-1} 3$ нечетный, то в перестановке вообще все циклы нечетные.

 Профиль  
                  
 
 Re: Putnam 2012 обобщение B6 - гипотезы
Сообщение06.12.2012, 19:51 
Заслуженный участник


08/04/08
8556
zhoraster в сообщении #655166 писал(а):
Только что дальше, не знаю. Единственное, что пока пришло в голову -- если не найдется такого $k$, что $3^k = -1\pmod {p-1}$, то перестановка четная. (Тогда циклы в перестановке очевидно разбиваются на пары.)
Я считал несколько перестановок явно: там встречаются и перестановки с длиной циклов равным 4. И вообще, думается, что длина цикла в общем случае не ограничена.
Я их много посчитать могу, но это, наверное, бесполезно.

(Оффтоп)

Я вообще не понимаю: каким боком тут четность цикла!? :facepalm: Меня поражает просто.

 Профиль  
                  
 
 Re: Putnam 2012 обобщение B6 - гипотезы
Сообщение06.12.2012, 19:57 
Модератор
Аватара пользователя


30/06/10
980

(Оффтоп)

Решил посмотреть, бывает ли такое (второе, что пришло в голову) вообще и нашел ошибку в OEIS, которая утверждает, что $\operatorname{ord}_{112}3=39$, это неверно: на самом деле $\operatorname{ord}_{112}3=12$. А я уже испугался, что что-то не так понимаю.


-- Чт дек 06, 2012 20:42:20 --

А насчет первого я соврал. (У меня почему-то и $p$, и $p-1$ простые.)

-- Чт дек 06, 2012 20:49:56 --

Ага, в одну сторону получилось вроде.

Пытаемся разбить циклы по парам: для $x\neq 0$
$$
x\to x^3\to x^9\to\dots
$$
и
$$
-x\to -x^3\to -x^9\to\dots
$$
Если не получилось разбить по парам, значит, для каких-то $x$ и $k\ge 1$ имеем $x^{3^k} = -x \pmod p$, то есть $x^{3^{k}-1} = -1 \pmod p$. Тогда $-1$ -- квадратичный вычет, то есть мы доказали... эмм... Необходимость.

 Профиль  
                  
 
 Re: Putnam 2012 обобщение B6 - гипотезы
Сообщение06.12.2012, 21:01 
Модератор
Аватара пользователя


30/06/10
980
Идея достаточности: в этом случае $-1$ -- квадратичный вычет. Пусть $i^2=-1\pmod p$. В этом случае разбиваем на пары так, как раньше. То, что не получилось разбить (то есть то, что "спаривается" с собой), разбиваем на пары
$$x\to x^3\to\dots\to -x \to -x^3\to\dots$$
и
$$ix\to -ix^3\to\dots\to -ix \to ix^3\to\dots$$
Вот теперь смелое утверждение, которое я не доказал и которое вообще, может быть, неверное: разобьется все, кроме цикла $i\to -i$. То есть перестановка нечетная.

 Профиль  
                  
 
 Re: Putnam 2012 обобщение B6 - гипотезы
Сообщение06.12.2012, 21:33 
Заслуженный участник


08/04/08
8556
zhoraster в сообщении #655178 писал(а):
Пытаемся разбить циклы по парам: для $x\neq 0$
$$ x\to x^3\to x^9\to\dots $$
и
$$ -x\to -x^3\to -x^9\to\dots $$
Если не получилось разбить по парам, значит, для каких-то $x$ и $k\ge 1$ имеем $x^{3^k} = -x \pmod p$, то есть $x^{3^{k}-1} = -1 \pmod p$. Тогда $-1$ -- квадратичный вычет, то есть мы доказали... эмм... Необходимость.
Не понял :-(
Вы рассматриваете варианты, в которых $\pi$ имеет $1$ или $2$ цикла? Просто $3$ - почти всегда не образующая $\mathbb{Z}_{p-1}^\times$ (это вообще циклическая группа только для простых Жермен), потому, даже если выделить циклы $(x \ x^3 \ x^9 \ ...)$ и $(-x \ -x^3 \ -x^9 \ ...)$, то это почти всегда будут не все циклы. И еще мне непонятно, как соотносятся четность перестановки и длина или количество ее циклов? Я думал, что почти никак.

zhoraster в сообщении #655209 писал(а):
Вот теперь смелое утверждение, которое я не доказал и которое вообще, может быть, неверное: разобьется все, кроме цикла $i\to -i$. То есть перестановка нечетная.
Тоже не уверен, что оно верно по причинам выше.

Я посчитаю несколько перестановок и тут выложу.

 Профиль  
                  
 
 Re: Putnam 2012 обобщение B6 - гипотезы
Сообщение06.12.2012, 23:32 
Модератор
Аватара пользователя


30/06/10
980
Четность перестановки и ее циклы связаны очень просто: четность перестановки совпадает с четностью количества четных циклов, которые она содержит.

Я не рассматриваю варианты, в которых у $\pi$ один или два цикла. И не рассматриваю варианты, является $3$ образующей или нет. Мне это не важно. Я разбиваю циклы на пары, причем все (не только какие-то два, которые я написал). И утверждаю, что, если $-1$ не является квадратичным вычетом, то у меня это получится. (Кстати, я доказал все же достаточность, это я все время их путаю.)

 Профиль  
                  
 
 Re: Putnam 2012 обобщение B6 - гипотезы
Сообщение07.12.2012, 02:23 
Модератор
Аватара пользователя


30/06/10
980
Кажись придумал. Давайте попробуем доказать необходимость по-честному (напомню, что доказал я достаточность).

Я буду писать подряд, что приходит в голову, с надеждой, что из этого получится решение. Поэтому заранее извиняюсь за беспорядок.

Итак, пусть $p=2\pmod 3$, $p=1\pmod 4$, то есть $p=12k+5$.

Вспомним, что группа $\mathbb Z_p^*$ циклическая. Пусть $p-1 = 2^k m$, $k\ge 2$, $m$ нечетно. Тогда $\mathbb Z_p^* \simeq \mathbb Z_{2^k}\times \mathbb Z_m $. Будем теперь считать циклы в последнем. Пусть $(a,b)\in  \mathbb Z_{2^k}\times \mathbb Z_m$ и $b\neq 0$. Рассмотрим цикл $(a,b)\to (3a,3b)\to \dots$ Его длина равна НОК длин циклов $a\to 3a\to \dots$ и $b\to 3b\to \dots$. Посмотрим на длину $c(a)$ первого.

Несложно доказать, что $$\operatorname{ord}_{2^n} 3  = \begin{cases} 2^{n-2}, &n\ge 3,\\ 2^{n-1}, &n=1,2.\end{cases}$$

Пусть $a\div 2^{k-r}$, $r\ge 3$. Имеем $a 3^{l} = a\pmod {2^k}\Leftrightarrow 3^{l} = 1\pmod {2^r}$. Отсюда $c(a)=2^{r-2}$. Таких $a$ $2^{r-1}$ штук и каждое из них входит в цикл длины $2^{r-2}$. То есть таких циклов четное количество, и они не влияют на четность перестановки.

Поэтому остались варианты: $a=0,2^{k-1},\pm 2^{k-2}$. Сейчас мы докажем, что с ними нечетное количество четных циклов. С первыми двумя все просто: цикл $(0,b)$ спариваем с циклом $(2^{k-1},b)$ и общее количество четное (NB в терминах $\mathbb Z_p^*$ это именно то самое спаривание цикла с противоположным).

Теперь пусть $b$ содержится в четном цикле $b \to 3b \to 3^2 b\to \dots$. Ему соответствуют два цикла с $a=\pm 2^{k-2}$:
$$
(2^{k-2},b) \to (-2^{k-2},3 b)\to (2^{k-2},3^2 b)\to\dots
$$
и
$$
(-2^{k-2},b) \to (2^{k-2},3 b)\to (-2^{k-2},3^2 b)\to\dots
$$
То есть общее количество таких циклов будет четное.

Если же $b$ -- нечетное, и $a = \pm 2^{k-2}$, то $(a,b)$ входит в четный цикл (просто потому, что цикл $a$ четный). Но количество нечетных циклов на $C_m$ нечетно, ведь $m$ нечетно! И каждому из них соответствует ровно один цикл с $a = \pm 2^{k-2}$. Значит, получается нечетное количество четных циклов. Поскольку до этого у нас было четное количество четных циклов, то общее количество нечетно, что и требовалось доказать.

Фух.

-- Пт дек 07, 2012 02:28:53 --

Кстати, в необходимости мы простоту $p$ полностью не использовали, только цикличность группы.

В достаточности она действительно нужна, потому что мы там на что-то сокращаем. (Хотя, может быть, можно как-то так же, как необходимость, доказать, не используя простоту $p$.)

 Профиль  
                  
 
 Re: Putnam 2012 обобщение B6 - гипотезы
Сообщение07.12.2012, 20:41 
Заслуженный участник


08/04/08
8556
zhoraster в сообщении #655292 писал(а):
Четность перестановки и ее циклы связаны очень просто: четность перестановки совпадает с четностью количества четных циклов, которые она содержит.

Я не рассматриваю варианты, в которых у $\pi$ один или два цикла. И не рассматриваю варианты, является $3$ образующей или нет. Мне это не важно. Я разбиваю циклы на пары, причем все (не только какие-то два, которые я написал). И утверждаю, что, если $-1$ не является квадратичным вычетом, то у меня это получится.
Ага! Я понял! Каждому циклу $c=(x \ x^3 \ x^9 \ ...)$ ставим в соответствие $c'=(-x \ -x^3 \ -x^9 \ ...)$. Пары $\binom{... \ i \ ... \ j \ ...}{... \pi(i) ... \pi(j) ...}$ переходят в $\binom{\ ... \ p-i \ ... \ p-j \ ...}{... p-\pi(i) ... p-\pi(j) ...}$ и тогда если $\pi(i)\star\pi(j)$, то $\pi(i)\overline{\star}\pi(j)$, где $\star\in\{<;>\}$, но $p-i<p-j$, значит по данной паре $(i;j)$ слагаемые, составляющие четность, совпадают и тогда, если циклы различны, то их произведение - четная перестановка. Остальное Вы написали явно :-)

Здесь $p$ можно заменить на $n$, $3$ - на $d$.

-- Пт дек 07, 2012 17:47:25 --

В обратную сторону завтра осилю...

 Профиль  
                  
 
 Re: Putnam 2012 обобщение B6 - гипотезы
Сообщение09.12.2012, 09:07 
Заслуженный участник


08/04/08
8556
Опять не понимаю :-(
Зачем Вы рассматриваете длины циклов?:
zhoraster в сообщении #655358 писал(а):
Его длина равна НОК длин циклов $a\to 3a\to \dots$ и $b\to 3b\to \dots$. Посмотрим на длину $c(a)$ первого.
zhoraster в сообщении #655358 писал(а):
Отсюда $c(a)=2^{r-2}$. Таких $a$ $2^{r-1}$ штук и каждое из них входит в цикл длины $2^{r-2}$. То есть таких циклов четное количество, и они не влияют на четность перестановки.
И как Вы ушли от сравнения $x^{3^t-1}\equiv -1\pmod p$?

Я пока пытаюсь решить аналогичным способом: разбиваем циклы на пары: $x\to x^3\to x^9\to...$ и $-x\to -x^3\to -x^9\to...$. Если эти циклы не совпадают, то их произведение - четная перестановка. Значит можно искать все $x:(\exists t) x^{3^t-1}\equiv -1\pmod p$ и разбивать их на классы по отношению $y=x^3$. Переходим от $\mathbb{Z}_p^\times$ к $\mathbb{Z}_{2^k}\times\mathbb{Z}_m, 2\nmid m$. Тогда каждому решению уравнения $x:(\exists t) x^{3^t-1}\equiv -1\pmod p$ биективно соответствует пара $(a,b)$: $3^ta\equiv 2^{k-1}+a\pmod{2^k}, 3^tb\equiv b \pmod m$ (элементу $-1$ соответствует пара $(2^{k-1},0)$).
Решаем 1-е:
$3^ta\equiv 2^{k-1}+a\pmod{2^k}$
$(3^t-1)a\equiv 2^{k-1}\pmod{2^k}$
$3^t-1=2^vq, q\nmid q, v<k$
$qa\equiv 2^{k-1-v}\pmod{2^{k-v}}$
$a=2^{k-1-v}a_1$
$qa_1\equiv 1\pmod 2, a_1 := q^{-1} \pmod 2$
$1\leqslant a\leqslant 2^k\Leftrightarrow 1\leqslant a_1\leqslant 2^{v+1}$ решений для заданного $v$, значит всего $\sum\limits_{v=1}^{k-1}2^{v+1}=2^{k+1}-2^2$ решений. Число $b$ подбирается из соотношения $m\mid b(3^t-1)$. А сколькими способами оно подбирается? А как это все потом на классы разбивать? :?

 Профиль  
                  
 
 Re: Putnam 2012 обобщение B6 - гипотезы
Сообщение10.12.2012, 20:10 
Модератор
Аватара пользователя


30/06/10
980
Sonic86 в сообщении #656084 писал(а):
Зачем Вы рассматриваете длины циклов?

zhoraster в сообщении #655292 писал(а):
Четность перестановки и ее циклы связаны очень просто: четность перестановки совпадает с четностью количества четных циклов, которые она содержит.


-- Пн дек 10, 2012 20:15:27 --

Sonic86 в сообщении #656084 писал(а):
И как Вы ушли от сравнения $x^{3^t-1}\equiv -1\pmod p$?

Дело в том, что теперь я почти не строю соответствий, просто считаю четные циклы. Это муторно, но осуществимо.

 Профиль  
                  
 
 Re: Putnam 2012 обобщение B6 - гипотезы
Сообщение10.12.2012, 20:18 
Заслуженный участник


08/04/08
8556
zhoraster в сообщении #656699 писал(а):
Sonic86 писал(а):
Зачем Вы рассматриваете длины циклов?

zhoraster писал(а):
Четность перестановки и ее циклы связаны очень просто: четность перестановки совпадает с четностью количества четных циклов, которые она содержит.
Ну! Вы же не рассматриваете четность циклов или их количество (т.е. количество рассматриваете, но кол-во каких циклов - четных или нечетных?). Вы только их длины рассматриваете...
Не понимаю :facepalm:

 Профиль  
                  
 
 Re: Putnam 2012 обобщение B6 - гипотезы
Сообщение10.12.2012, 20:20 
Модератор
Аватара пользователя


30/06/10
980
Четный цикл - это цикл четной длины, прошу прощения за некоторую вольность в терминологии.

В принципе, меня интересует больше не длина, а ее четность. Или для какой-то длины я доказываю, что циклов такой длины четное количество (и они потому не влияют на четность перестановки).

 Профиль  
                  
 
 Re: Putnam 2012 обобщение B6 - гипотезы
Сообщение10.12.2012, 20:22 
Заслуженный участник


08/04/08
8556
zhoraster в сообщении #656707 писал(а):
Четный цикл - это цикл четной длины, прошу прощения за некоторую вольность в терминологии.
В смысле??? :shock: Четный цикл - это цикл из $A_n$, т.е. цикл, являющийся произведением четного числа транспозиций! $(132)$ - четный цикл, $(12)$ - нечетный.
Я для них статистику строил - в пределах $n\leqslant 80$ свободных от квадратов все правильно. Могу и дальше подсчитать...

 Профиль  
                  
 
 Re: Putnam 2012 обобщение B6 - гипотезы
Сообщение11.12.2012, 14:23 
Модератор
Аватара пользователя


30/06/10
980
Sonic86, ну я так назвал, что поделать. (И даже извинился.) В Вашем смысле четный цикл - цикл нечетной длины, а нечетный цикл - цикл четной длины.

 Профиль  
                  
Показать сообщения за:  Поле сортировки  
Начать новую тему Ответить на тему  [ Сообщений: 15 ] 

Модераторы: Модераторы Математики, Супермодераторы



Кто сейчас на конференции

Сейчас этот форум просматривают: нет зарегистрированных пользователей


Вы не можете начинать темы
Вы не можете отвечать на сообщения
Вы не можете редактировать свои сообщения
Вы не можете удалять свои сообщения
Вы не можете добавлять вложения

Найти:
Powered by phpBB © 2000, 2002, 2005, 2007 phpBB Group